Naked Science Forum

General Science => General Science => Topic started by: Aeris on 26/10/2021 20:46:34

Title: Even More Questions that Antimatter to Me
Post by: Aeris on 26/10/2021 20:46:34
Awhile ago, I posted a list of questions on this site about Antimatter and it's various properties. I was extremely satisfied with the answers I got, but there's still so much more about Antimatter that I still don't fully understand. Here's three new questions I have about Antimatter.

1. As just about everyone who’s familiar with antimatter knows, whenever antimatter comes into contact with regular matter, they both annihilate each other in a burst of energy. Why exactly does this happen though? Antimatter is identical to regular matter in just about every conceivable way, with the sole difference being its electric charge. How does this one, little thing cause antimatter to react so violently with regular matter?
2. A lot of people say that there is a lot more matter in our universe than antimatter, but how certain can we really be about this? Like, are we absolutely certain that there is way more regular matter in our universe than antimatter? What’s wrong with the idea of the universe having equal amounts of both, albeit in places far apart from each other?
3. Let’s say, hypothetically speaking (key word, hypothetically, NOT theoretically), I created a wooden, regular matter chair from a huge amount of energy without creating any antimatter whatsoever. It really doesn’t matter (no pun intended) how exactly I did this. All that matters is that I created lots of matter without creating even a little bit of antimatter. How many physical laws would I break from doing this? I created a ton of electrically neutral molecules, and since conservation of charge demands only that the net electric charge of the entire universe is 0, I have to assume that that law is safe for the most part. What about the other laws though? Which ones would render this imaginary process impossible, both in theory and in practice?
Title: Re: Even More Questions that Antimatter to Me
Post by: Origin on 26/10/2021 21:38:51
Awhile ago, I posted a list of questions on this site about Antimatter and it's various properties. I was extremely satisfied with the answers I got,
Good.
Antimatter is identical to regular matter in just about every conceivable way, with the sole difference being its electric charge.
That statement is incorrect.  No one said that in response to your earlier questions.
Why ask more questions when you seem to have ignored the answers to the last set?
Title: Re: Even More Questions that Antimatter to Me
Post by: Halc on 26/10/2021 21:50:28
As just about everyone who’s familiar with antimatter knows, whenever antimatter comes into contact with regular matter, they both annihilate each other in a burst of energy.
But they always show something like an electron/positron collision, which leaves nothing but photons. But it's not always the case. What if a positron meets a free neutron? Hard to even find a site describing that, but they're not going to annihilate each other.
A normal rock meets an antimatter rock and sure, it goes bang, but most of the material in both will survive and just be moving away fast.

Quote
Why exactly does this happen though? Antimatter is identical to regular matter in just about every conceivable way, with the sole difference being its electric charge.
No, it is not just a difference in charge, as was mentioned before. Yes, a charged particle's antiparticle has the opposite charge, but plenty of matter has no charge at all. Electrons and protons have opposite charge but neither is antimatter.  A neutron and antineutron do not differ in charge.

Quote
How does this one, little thing cause antimatter to react so violently with regular matter?
A small mass is equivalent to considerable energy, so when mass in annihilated, the resulting energy has to be proportional. The Hiroshima bomb converted less than a gram of matter into energy.

Quote
A lot of people say that there is a lot more matter in our universe than antimatter, but how certain can we really be about this?
We can't, but we can observe the lack of interaction at boundaries that would exist if the clumps were smaller than the observable universe. We don't see that, so if the antimatter is out there, it's further away than we can see.
Quote
What’s wrong with the idea of the universe having equal amounts of both, albeit in places far apart from each other?
That works.

Quote
Let’s say, hypothetically speaking (key word, hypothetically, NOT theoretically), I created a wooden, regular matter chair from a huge amount of energy without creating any antimatter whatsoever. It really doesn’t matter (no pun intended) how exactly I did this. All that matters is that I created lots of matter without creating even a little bit of antimatter. How many physically laws would I break from doing this?
Don't know. There's not a conservation law. I can dump mostly antimatter into a black hole and it's just gone, and it eventually fizzles away in Hawking radiation which is neither matter nor antimatter, thus illustrating the lack of conservation law. Still, I know of no way to create one without the other, unless I somehow sort it out and get rid of one in some way.
Look at the bomb I mentioned. It converted matter to energy without requirement of equivalent antimatter. Just do that in reverse. The laws of physics are time-symmetrical, so there's actually nothing illegal about the chair you made.

Quote
I created a ton of electrically neutral molecules, and since conservation of charge demands only that the net electric charge of the entire universe is 0
Black holes preserve charge, so yes on this. Charge is indeed a conserved quantity, but your chair didn't have a net charge, so it doesn't violate conservation laws.
Title: Re: Even More Questions that Antimatter to Me
Post by: evan_au on 26/10/2021 22:14:09
Quote from: OP
1. Antimatter is identical to regular matter in just about every conceivable way,
I would prefer to say "Antimatter is the opposite of regular matter in just about every conceivable way"- except probably gravitational attraction (and perhaps the direction that it travels through time).

As I recall, it was Feynman made the mind-bending comment that a positron is just an electron traveling backwards through time (a time-reversed & charge-reversed electron).
See: https://en.wikipedia.org/wiki/CPT_symmetry

Quote
2. What’s wrong with the idea of the universe having equal amounts of both, albeit in places far apart from each other?
That is still possible, but generally considered unlikely: On the boundary of matter-dominated space and anti-matter space you would expect a flood of gamma rays. This has not been seen.
- There is no known mechanism at the Big Bang that could separate matter & antimatter before they could annihilate
- but if gravitational attraction is different between matter and antimatter, that might do it. This experiment is still underway at the LHC, but with no accurate results as yet.

Quote
3. Which ones would render (creation of matter only) impossible, both in theory and in practice?
It's true that forming a neutral atom from electromagnetism obeys conservation of charge.
- But there are other quantities that are conserved in quantum theory (at least approximately)
- Conservation of baryon number: You have not created an equal number of protons and anti-protons
- Conservation of lepton number: You have not created an equal number of electrons and anti-electrons (positrons)

There are instances where these conservation rules are occasionally violated (specifically, via the Weak Nuclear Force), but no scenarios have been found that can account for the observed ratio of matter to antimatter
- or indeed, the ratio of (matter+antimatter) to electromagnetism: If matter & antimatter were created in equal quantities at the Big Bang, it is expected that they would annihilate before they could separate, leaving far more electromagnetism in the universe than we observe today.

See: https://en.wikipedia.org/wiki/Conservation_law#Approximate_laws

Oops! Overlap with Halc...
Title: Re: Even More Questions that Antimatter to Me
Post by: Aeris on 27/10/2021 18:41:01
Sorry everyone for the late reply. Had a very busy day. Gonna go through this one by one.

Origin

"That statement is incorrect.  No one said that in response to your earlier questions.
Why ask more questions when you seem to have ignored the answers to the last set?"

Care explaining to me why this statement is wrong? Just saying a statement is incorrect without actually explaining WHY it's incorrect holds about as much water as religious people saying God exists because they said he does (which is why I ended up dropping my religion years ago). Also, this statement has absolutely nothing to do with my old questions, it's related to an entirely new question.

Halc

"But they always show something like an electron/positron collision, which leaves nothing but photons. But it's not always the case. What if a positron meets a free neutron? Hard to even find a site describing that, but they're not going to annihilate each other. A normal rock meets an antimatter rock and sure, it goes bang, but most of the material in both will survive and just be moving away fast."

Hmmmm... that's actually really interesting. So an antimatter particle can only annihilate its regular matter counterpart, and not just any random regular matter particle? Does that mean that antimatter atoms and molecules won't react violently with regular matter atoms and molecules? If so, could that mean there's entire planets and galaxies out there in the universe made of antimatter that's safe from annihilation?

"No, it is not just a difference in charge, as was mentioned before. Yes, a charged particle's antiparticle has the opposite charge, but plenty of matter has no charge at all. Electrons and protons have opposite charge but neither is antimatter.  A neutron and antineutron do not differ in charge."

Ok... that still doesn't answer my question though. WHY does regular matter and antimatter annihilate each other upon contact?

"We can't, but we can observe the lack of interaction at boundaries that would exist if the clumps were smaller than the observable universe. We don't see that, so if the antimatter is out there, it's further away than we can see."

But like, would the energy bursts produced from those reactions REALLY be that noticeable to us? We can only observe a small percentage of the universe, and some have theorized that it's infinite in size.

evan_au

"It's true that forming a neutral atom from electromagnetism obeys conservation of charge.
- But there are other quantities that are conserved in quantum theory (at least approximately)
- Conservation of baryon number: You have not created an equal number of protons and anti-protons
- Conservation of lepton number: You have not created an equal number of electrons and anti-electrons (positrons)"

What do you mean approximately? Either those quantities are conserved, or those quantities are NOT conserved. I've never heard of any theory that would try to have it both ways (or neither of them).

Also isn't baryon number and lepton number already violated simply on the virtue of the universe having (at least, to our knowledge) more matter (protons and electrons) than antimatter (antiprotons and positrons)? Even if that didn't do it though, surely things such as nuclear explosions, nuclear fusion, hawking radiation and proton decay (assuming it turns out to be real) violate these numbers?

Thanks everyone for your answers :)   

 
   


Title: Re: Even More Questions that Antimatter to Me
Post by: Halc on 27/10/2021 21:56:00
So an antimatter particle can only annihilate its regular matter counterpart, and not just any random regular matter particle? Does that mean that antimatter atoms and molecules won't react violently with regular matter atoms and molecules?
The one is protons, neutrons and electrons and such, and the antimolecules are composed of anti-all-those-things, so they'll annihilate just fine if you can get them to get close enough. So an anti-rock sitting on Earth is going to completely blow up, taking its mass in regular matter with it, because the parts have nowhere to go that doesn't just meet more matter. But two rocks in space will band and just end up moving away from other in a hot expanding cloud of dust that won't interact hardly at all.

Quote
If so, could that mean there's entire planets and galaxies out there in the universe made of antimatter that's safe from annihilation?
If they're safe its because there's no regular matter nearby. There has to be a border where they meet somewhere, and you'd be able to see that. We don't see it, so if there is such a boundary, it's outside the visible universe.
Quote
But like, would the energy bursts produced from those reactions REALLY be that noticeable to us?
They probably wouldn't be bursts, but a continuous line of radiation. Yes, they'd be very visible, especially in the cosmic microwave background which would clearly show boundaries.

Quote
Ok... that still doesn't answer my question though. WHY does regular matter and antimatter annihilate each other upon contact?
It just does. There doesn't seem to be a why to it. That's what is observed, and science describes the behavior, and doesn't posit the reason for the behavior. There might be sites that flesh out a better answer than mine. Hey, I'm hardly a particle physicist here.
You create antimatter and it is stupidly difficult to prevent it from destruction immediately. They've done it, slowing it down to actually create anti-hydrogen molecules.

The sun creates antimatter (mostly positrons) at a pretty furious rate as part of the process of the creation of neutrons it needs for fusion reactions. Each created antimatter particle lives probably less than a nanosecond on average.
Title: Re: Even More Questions that Antimatter to Me
Post by: Aeris on 28/10/2021 09:31:07
Quote from: Halc
If they're safe its because there's no regular matter nearby. There has to be a border where they meet somewhere, and you'd be able to see that. We don't see it, so if there is such a boundary, it's outside the visible universe.
But would you still be able to see it even if it was somewhere outside of the observable universe? The light from countless stars and galaxies is still able to reach our eyes for quite a long time even, though those planets and galaxies are far beyond us (so far beyond us that, even with hypothetical light-speed technology, we may still not ever be able to visit them).

Quote
It just does. There doesn't seem to be a why to it. That's what is observed, and science describes the behavior, and doesn't posit the reason for the behavior. There might be sites that flesh out a better answer than mine. Hey, I'm hardly a particle physicist here.
You create antimatter and it is stupidly difficult to prevent it from destruction immediately. They've done it, slowing it down to actually create anti-hydrogen molecules.

The sun creates antimatter (mostly positrons) at a pretty furious rate as part of the process of the creation of neutrons it needs for fusion reactions. Each created antimatter particle lives probably less than a nanosecond on average.
So... we know that matter and antimatter annihilate each other upon contact, but we do know WHY they do that? That's... very lame honestly. Also, why is it so difficult to prevent antimatter from immediately interacting and annihilating regular matter? Are they electrically attached to each other or something?

I actually didn't know that the Sun created antimatter particles. I especially didn't know that it NEEDS them for its constant nuclear fusion reactions. How exactly do they contribute to the Sun's energy generating reactions exactly? I'm curious to know.
Title: Re: Even More Questions that Antimatter to Me
Post by: Halc on 28/10/2021 13:49:47
But would you still be able to see it even if it was somewhere outside of the observable universe?
If you can see it, it is by definition inside the observable universe.

Quote
The light from countless stars and galaxies is still able to reach our eyes for quite a long time even, though those planets and galaxies are far beyond us (so far beyond us that, even with hypothetical light-speed technology, we may still not ever be able to visit them).
The observable universe is all locations in cosmological space at which a cause might have occurred which possibly had an effect on us now. This is far more than we can see. The observable radius is currently about 48 BLY. The maximum proper distance from us of any light that has ever reached here only has a radius of under 6 BLY. That's the maximum proper width of our past light cone in cosmological coordinates. The CMB is pretty much the furthest we can see with visible light, and that was emitted from a proper distance of perhaps 2 million LY away.

What you're talking about seems to be the event horizon, beyond which light emitted cannot reach Earth even given infinite time, and beyond which we cannot visit even given the fastest ship possible. That's currently about 16 BLY (proper) distance away, and is only there due to acceleration of expansion. If space expansion was not accelerating, there would be no event horizon at all and light/ships from here could reach any location in space given sufficient time.

Quote
So... we know that matter and antimatter annihilate each other upon contact, but we do know WHY they do that? That's... very lame honestly.
I said I don't know. It seems similar to asking WHY a positively charged particle is attracted to a negatively charged one. No matter how good the explanation, you can always ask 'why' to go a level deeper.

Quote
Also, why is it so difficult to prevent antimatter from immediately interacting and annihilating regular matter?
You try to prevent a fast moving object in the middle of other matter from touching literally anything. It has been done, but only with incredible effort.

Quote
I actually didn't know that the Sun created antimatter particles. I especially didn't know that it NEEDS them for its constant nuclear fusion reactions.
Yes. A hydrogen nucleus is just a proton, and helium needs two protons and two neutrons, and the universe didn't create many free neutrons. So two of the protons need to be changed into neutrons first, and that process creates positrons to preserve the charge.
(https://upload.wikimedia.org/wikipedia/commons/thumb/8/85/Fusion_in_the_Sun.svg/250px-Fusion_in_the_Sun.svg.png)
At the top you see two protons colliding, producing a deuterium nucleus (a proton and neutron) plus the positron and a gamma ray.

For the record, there is also the CNO cycle that is a catalyst reaction more common in larger stars:
(https://upload.wikimedia.org/wikipedia/commons/thumb/2/21/CNO_Cycle.svg/300px-CNO_Cycle.svg.png)
Exact same net effect. Using clock coordinates, Helium comes out at 11 O Clock. Two positrons are created during spontaneous decay of the respective unstable nuclei, one each at 3 and 9 O Clock respectively.

Quote
How exactly do they contribute to the Sun's energy generating reactions exactly? I'm curious to know.
Well a positron is going to find an electron very quickly and turn into high energy, so I think that counts as contributing to the energy.
Title: Re: Even More Questions that Antimatter to Me
Post by: Eternal Student on 28/10/2021 14:26:20
Hi.

I'm late to join this thread and it seems like you (all of you) have already sorted out a lot of ideas.

    Let's start with Q.1.
1. As just about everyone who’s familiar with antimatter knows, whenever antimatter comes into contact with regular matter, they both annihilate each other in a burst of energy. Why exactly does this happen though? Antimatter is identical to regular matter in just about every conceivable way, with the sole difference being its electric charge. How does this one, little thing cause antimatter to react so violently with regular matter?
    Firstly, anti-matter doesn't always interact with matter.   I think @Halc made this point first.  I'm just providing some more detail and/or re-phrasing the ideas.
   Annihilation requires a particle to meet it's own exact anti-particle partner.   So an anti-proton cold annihilate with an ordinary proton but if an anti-proton came into close proximity with some other particle of ordinary matter like a neutrino than nothing very much is likely to happen.  The anti-proton is one of a class of particles called Baryons while the neutrino is a different class of particle, a Lepton.  They can't be particle and anti-particle partners.
    I took a Lepton as one of my particles in a collision for the first example because they are quite fundamental particles,  the situation gets more complicated for particles that are composite particles.
   
    What really happens when a proton collides with it's exact anti-particle partner?
   The proton is not fundamental: It contains an evolving mixture of quarks, antiquarks and gluons. These quarks and gluons are collectively called partons and are held together by the QCD interaction, like marbles in a bag. While the marble metaphor is flawed in many ways, like most real-world imaginings of quantum behavior, it provides a helpful analogy to consider the various possible outcomes of particle collisions, so let’s go with it. 
(https://www.fnal.gov/pub/today/archive/archive_2014/images/fsr-dzero-06022014-figure01-s.jpg)
   The most common outcome of a proton-antiproton collision is that the two hadrons simply break apart — the two bags of marbles break — weakly scattering the internal quarks and gluons. This is called a “soft” interaction.

[Taken from a Fermilab article:  https://news.fnal.gov/2014/02/seeing-double-in-proton-antiproton-collisions]
   If you read the rest of the short article you'll see that they are implying that quite often you get a shower of smaller particles of various types and only occasionally is there a direct interaction between two partons where new particles could emerge which may include something like an annihilation event where only photons emerge.

    This is quite different to the way most Science Fiction and popular Science articles describe matter and anti-matter reactions.  They give the impression that if you got an anti-particle anywhere near it's ordinary matter equivalent they would always and completely annihilate.  This is simply not the situation we observe, most interactions between protons and anti-protons are much "softer" than this.   However, in our ordinary everyday lives we have lots of ordinary matter around us.  So that if some anti-particle was dropped into an ordinary room in your house, then it can have millions of encounters with ordinary matter particles in a fraction of a second and then (by probability alone) it should interact.  It's not guaranteed that it interacts with the first particle of ordinary matter it encounters, the whole process is much more random than this.  However, within a very short amount of time we would reasonably expect to have nothing but some photons left behind.

    There are also some interesting structures that might exist, albeit very briefly, and involve a particle being in close vicinity of its anti-particle without annihilating:   https://en.wikipedia.org/wiki/Positronium. 

    Let's focus on the next part of the original question "Why exactly does this (annihilation) happen though?".   It doesn't seem that there is any "law" or imperative that forces annihilation's to happen when particle and anti-particle pairs come into contact.  It is likely to be more of a random process that can happen rather than something that must happen.  It's possible to imagine the phenomena on a macroscopic scale:  A random process like this should be governed by thermodynamics and statistical mechanics.   In the current state of the universe the density of photons is low and most of these photons are not of high energies.  So it is statistically likely and thermodynamically favourable for particle and anti-particles to annihilate and produce high energy photons.  This produces a higher entropy state.  This change may be so favourable that we almost always observe particle and anti-particle annihilation's and almost never observe a reverse process.  Meanwhile, we can imagine that in the early universe the situation was different, we had a higher density of photons and the average energy of these photons was also much higher.  In the early universe the reverse process (particle and anti-particle creation from photons) might have been thermodynamically favoured.   So, this macroscopic view is one way to explain WHY anti-particles tend to annihilate with their ordinary particle partners:  It's just random but now so thermodynamically or statistically favoured that we will almost always observe this process and not the reverse and the change is rapid when it happens.   [see video reference 2]

   The later part of the question states the following:   "Antimatter is identical to regular matter in just about every conceivable way, with the sole difference being its electric charge".    This has been adequately discussed by @Halc and @evan_au.   Every particle in the standard model of particle physics should have a corresponding anti-matter particle.  It's a bit messy since some particles may be their own anti-particle.
(https://upload.wikimedia.org/wikipedia/commons/thumb/a/a3/Standard_Model_of_Elementary_Particles_Anti.svg/525px-Standard_Model_of_Elementary_Particles_Anti.svg.png)
[Image taken from Wikipedia]
   There should be 61 fundamental particles if you include all the anti-particles.   These can combine to form various composite particles like protons (or entire atoms) and if all the fundamental particles in these composites are replaced with their anti-particle partners then the new composite particle is generally called an anti-proton (or anti-atom respectively).
  To the best of my knowledge we haven't observed the anti-particle of every fundamental particle and so the idea remains theoretical.  However, we are using anti-electrons  (positrons) routinely in medical P.E.T. scanners and CERN seem able to collect anti-protons and anti-electrons to construct complete anti- Hydrogen atoms.  So at least some anti-particles do seem to exist and behave as we expect.
   While Popular Science does describe anti-particles as being exactly the same as ordinary matter but with an opposite electrical charge, this is not really a sufficient characterisation of anti-particles.   It's true that anti-matter would have an opposite charge but many particles have 0 charge anyway and then -0 and +0 are just the same thing.  A better description of an anti-matter particle is that it should have the opposite charge AND behave like a mirror version under the Weak Force [see video reference 1].     The implication being that it behaves exactly the same under gravity, strong force and the electromagnetic force (noting that charge is negated).  So in this sense it behaves as we would expect ordinary matter to behave and all we can really notice is that the charge is opposite since most of us do not have the equipment in our house to probe and test a particle for interaction under the weak force.  If you did have that ability then anti-particles interact with W bosons to produce anti-particle equivalents of the same interaction between the W boson and its ordinary matter partner.   As previously stated, this remains theoretical and CERN are currently conducting experiments to try and determine if their anti-Hydrogen atoms really do respond to a gravitational field in the same way as we would expect an ordinary Hydrogen atom to respond.

This is an easily followed YT video about anti-matter.  I'm a bit too lazy to find more authoritative references for some of the things I've mentioned above (and also fairly sure that most readers wouldn't want to try reading more formal texts on the subject anyway).
Why is there more anti-matter in the universe?   by the Royal Institute, released on You Tube.  Duration approx. 7 minutes.

References:   [1]    at   0:40  to  1:00     Description of anti-particles.
[2]   at 1:40 to 2:30    A brief description of annhilation and particle synthesis as a reversible or equilibrium process in the early universe.


Best Wishes.
Title: Re: Even More Questions that Antimatter to Me
Post by: Aeris on 29/10/2021 10:01:34
Halc

"If you can see it, it is by definition inside the observable universe."

"The observable universe is all locations in cosmological space at which a cause might have occurred which possibly had an effect on us now. This is far more than we can see. The observable radius is currently about 48 BLY. The maximum proper distance from us of any light that has ever reached here only has a radius of under 6 BLY. That's the maximum proper width of our past light cone in cosmological coordinates. The CMB is pretty much the furthest we can see with visible light, and that was emitted from a proper distance of perhaps 2 million LY away."

"What you're talking about seems to be the event horizon, beyond which light emitted cannot reach Earth even given infinite time, and beyond which we cannot visit even given the fastest ship possible. That's currently about 16 BLY (proper) distance away, and is only there due to acceleration of expansion. If space expansion was not accelerating, there would be no event horizon at all and light/ships from here could reach any location in space given sufficient time."

Yeah, ignore what I said yesterday about that. I had misinterpreted what was said in this video
 
"I said I don't know. It seems similar to asking WHY a positively charged particle is attracted to a negatively charged one. No matter how good the explanation, you can always ask 'why' to go a level deeper."

I'm not so sure that kind of logic can really be applied to something like this. Like, say for a moment that matter and antimatter reactions are the result of the electromagnetic force doing some funky opposites attract junk or whatever (this is not at all the answer, but pretend for the sake of this response that it is). Where can you really go from there? I guess you could ask how the electromagnetic force came into existence in the first place, but at that point, you've pretty much got to accept the notion of either emergence from nothing or eternal existence.

Eternal Student

So... it's a case of probability then?

Title: Re: Even More Questions that Antimatter to Me
Post by: alancalverd on 29/10/2021 10:50:17
"I said I don't know. It seems similar to asking WHY a positively charged particle is attracted to a negatively charged one. No matter how good the explanation, you can always ask 'why' to go a level deeper."
You should never ask "why" in physics. The question presupposes a controlling  entity with an ulterior motive, for which there is no evidence.
Physics (and indeed all sciences, since all others are more or less complicated embodiments of physics) is about observing "what" happens and building a predictive mathematical model of "how" it happens. We take the universe at face value, though we scrutinise that face very carefully.
We observe that some particles are attracted to one another and that attraction is modelled by the mathematics we call electrostatics, which is based on the conservation of complementary charges. These are arbitrarily* labelled positive and negative and have been found to be quantised in units of + or - 1 electron charge.
Any "deeper" explanation is a fairy story.


*The definition, established hundreds of years ago, is that rubbing ebonite with cat fur gives a positive charge to the fur and a negative charge to the ebonite. No kidding - it's still the basis of electrostatics.
Title: Re: Even More Questions that Antimatter to Me
Post by: evan_au on 29/10/2021 11:17:51
Quote from: Aeris
What do you mean approximately? Either those quantities are conserved, or those quantities are NOT conserved.
Wikipedia has a subcategory of approximate conservation laws..
https://en.wikipedia.org/wiki/Conservation_law#Approximate_laws

The Weak Nuclear Force can (sometimes) violate some conservation laws that are obeyed by the other fundamental forces. But the cross-section of the Weak Force is so small that these events are rather rare.

Quote
WHY does regular matter and antimatter annihilate each other upon contact?
As Eternal Student suggested, I suspect it has a lot to do with entropy.

If we think of the "Heat Death of the Universe" as being the highest entropy state, with all energy spread thinly through a low temperature universe...
- An isolated electron or positron represents a concentrated local mass/energy of around 0.5 MeV/c2, unaffected by the expansion of the universe.
- If an electron and positron collide in space and produce a pair of gamma rays, these will be eventually red-shifted away to lower energy by the general expansion of the universe
- Even faster, if an electron and positron collide in the core of a star, the pair of 0.5 MeV gamma rays experience many collisions, breaking down the high energy into many lower energy photons, until they are eventually emitted at the surface of the star as many photons with perhaps 1eV energy, travelling in 500,000 different directions. This has even higher entropy (and these, too will be red-shifted by the expansion of the universe).

Quote
why is it so difficult to prevent antimatter from immediately interacting and annihilating regular matter?
The LHC has a bottle to store anti-protons and positrons, using electric fields in an extreme vacuum.

Even trickier, they also have a bottle that can store neutral anti-hydrogen, using powerful superconducting magnets.
https://home.cern/science/physics/antimatter/storing-antihydrogen
Title: Re: Even More Questions that Antimatter to Me
Post by: Eternal Student on 29/10/2021 22:52:48
Hi again.
Quote from: Halc
"If you can see it, it is by definition inside the observable universe."
    That's true, you just have to be careful about what you meant by being able to see it.    You may not be able to see distant objects as they are NOW.

Quote from: Halc
"The observable radius is currently about 48 BLY. The maximum proper distance from us of any light that has ever reached here only has a radius of under 6 BLY. That's the maximum proper width of our past light cone in cosmological coordinates.
   Yes to the first and not quite to the second.   6 units might be the width of a cone in cosmological co-ordinates but it wouldn't be BLY as those units.   By cosmological co-ordinates you're probably talking about co-moving co-ordinates and those have distances in fairly arbitrary units.   Well.... they aren't entirely arbitrary,  1 unit of distance in those co-ordinates would be equal to a fixed number of light years (let's say 1 BLY) provided you kept time frozen.   I'm not usually able to freeze time, so then 1 unit of spatial separation in the co-moving frame is a time-dependant, ever increasing amount of actual physical distance.   (Minor note,  I'm finding more references claiming the radius of the observable universe is approx. 46.5 BLY  and not 48 BLY - but it's not worth quibbling over).

Quote
  (from Aeris)
(...Concerning previous discussion of anti-matter and matter annihilation....)
So... it's  a case of probability then?
   Well, it does seem to be.    However, you shouldn't just believe everything I've said.  The earlier post pulls a collection of different articles and ideas together.   It's an attempt to answer a direct question   "Why does matter annihilate with anti-matter?"   using what information is available.  You should check the sources of information yourself, make your own decisions and carefully note where any caveats or direct menton of a lack of evidence has been mentioned.  For example, I mentioned that although there should be an anti-particle for every fundamental particle in the standard model of physics, we haven't observed all of these and the idea does remain theoretical.
   As Alancalverd indicated, many people don't ask why and indeed there aren't any articles I can find that attempt to explain why.  So what you've got is a collection of things we do know about related situations and these have been stitched together to try and answer your question in what is hopefully a logical and consistent way.   It's the best I can offer:  There just aren't any definitive articles or texts written to explain WHY it happens (well, not that I could find) although there are many theories in which it CAN happen.   Presumably you haven't found it yourself with Google and/or you found quite a lot of conflicting information.

1.   The first thing to note should be that anti-matter just doesn't always annihilate with its ordinary matter partner when they come into contact.  That's a myth, Science Fiction or PopSci  idea.  Check reports from Fermilab about proton and anti-proton collisions (and there are also other establishments that work with anti-protons).

2.  From this we can reasonably conclude that there is no hard or immutable rule in nature that says matter must always annihilate with anti-matter when they come into contact.   It was obviously going to be complicated when an anti-matter particle like an anti-Neutron came into contact with an ordinary matter particle like a proton because these two composite particles do share some common quarks, so that some partial annihilation could have been possible but total annihilation wasn't.  However, we don't even have to consider anything that complicated.  Fermilab have been colliding protons into their direct anti-proton partners and still they didn't always undergo total annihilation.
     Halc mentioned that most articles only consider the most elementary particles like electrons and positrons annihilating.  However, if you dig hard enough you'll see that even these interactions don't seem to happen all the time.  In a hospital P.E.T. scanner it is the mainstream theory that positrons emitted by the radio-tracers in the patient do travel a short distance through the tissue before annihilation with electrons.  Now it should have gone past or indeed right through quite a few (thousand) orbital clouds of electrons around atoms during this travel. 
   ...The emitted positron travels in tissue for a short distance (typically less than 1 mm, but dependent on the isotope[51]), during which time it loses kinetic energy, until it decelerates to a point where it can interact with an electron.[52]   -  taken from https://en.wikipedia.org/wiki/Positron_emission_tomography
    From this we can conclude that even positrons and electrons don't always annihilate when they are in close proximity and we can already see that there are going to be problems deciding when they are "in contact" since everything at this scale is governed by quantum mechanics and particles only have a certain probability of being found at a certain place.  The PET scanner also indicates that velocity might be one factor that influences the likelihood of an annihilation event occurring.

 --- I've deleted the rest of what I was going to say, I've already said too much.  Let's just wrap this up:  The annihilation of particles and their anti-particles is not a certainty just because they are in close proximity.  It may be genuinely random or there may be a collection of factors that would entirely determine if the interaction occurrs.  However, if there is such a collection of factors it is currently not well understood and not realistically in our control, so that on a macroscopic scale,  annihilations are effectively a random process even if they are not a genuinely random process.  This is sufficient to apply some probability theory and thermodynamics and statistical mechanics follows (as in the earlier post).

Best Wishes.

LATE EDITING:  Fixed errors in the quotes from other people.  Most are from Aeris but some were incorrectly identified as being from other people.
Title: Re: Even More Questions that Antimatter to Me
Post by: alancalverd on 29/10/2021 23:34:07
That's true, you just have to be careful about what you meant by being able to see it.    You may not be able to see distant objects as they are NOW.
Indeed if you take a classical view of information, you can't ever see anything as it is, but only as it was when the light left it.
Title: Re: Even More Questions that Antimatter to Me
Post by: evan_au on 30/10/2021 00:23:43
Quote from: Eternal Student
even positrons and electrons don't always annihilate when they are in close proximity..
An electron and positron frequently form "positronium" for a short period before they annihilate.
- So annihilation is much more likely if the electron and positron are in the "same" location and velocity for some time before they annihilate.

The average lifetime of Positronium depends on the spins of the electron and positron:
- Parallel spins: 0.12 ns (nanoseconds)
- Opposite spins: 140 ns
- So the lifetime varies by 3 orders of magnitude, depending on the spins (ie their magnetic fields)

A medical PET scanner typically generates positrons with an average energy of 0.9MeV, so they are travelling at relativistic speeds. At these speeds, the positron is not near any particular electron long enough for annihilation to be likely with that electron.
It takes many elastic collisions with electrons and atomic nuclei (emitting electromagnetic radiation) before the positron slows down enough to form positronium, after which it will annihilate with confidence.
See: https://en.wikipedia.org/wiki/Positronium
Title: Re: Even More Questions that Antimatter to Me
Post by: Halc on 30/10/2021 00:43:19
LATE EDITING:  Fixed errors in the quotes from other people.  Most are from Aeris but some were incorrectly identified as being from other people.
But you still got it wrong, and I edited your post, putting my name on the words that were mine.
A good deal of the problem is that Aeris doesn't seem to be using the quote feature and is just putting other people's works in quotation marks which doesn't identify whose words they are.

BTW, great post, especially the part of how much trouble an anti-proton might have in its quest to be totally annihilated. It's far more informative than my answer since I was unaware of how precise an interaction was required for a full reaction, even if in stages.

If it's that much work do destroy an antiproton (negatron?), imagine how much work it is to put one together. I admire the guys that manage not only to do it, but to capture it as well. Now they have to make anti-helium, or better, anti-water, which seems easier to contain as rocket fuel. You can even store it in a nice big anti-fuel-tank and just figure out how not to touch the tank. Plumbing fittings are going to be interesting. You thought it was hard going from steel to copper pipes....

Quote
Quote from: Halc
If you can see it, it is by definition inside the observable universe."
That's true, you just have to be careful about what you meant by being able to see it. You may not be able to see distant objects as they are NOW.
Indeed, and it has nothing to do with objects being distant. I cannot see my own keyboard as it is NOW. It's just a super-recent past. Observable universe is a relation between the current event (here and now) and points in space per cosmological coordinates.

Quote
Quote from: Halc
"The observable radius is currently about 48 BLY. The maximum proper distance from us of any light that has ever reached here only has a radius of under 6 BLY. That's the maximum proper width of our past light cone in cosmological coordinates.
Yes to the first and not quite to the second.
6 units might be the width of a cone in cosmological co-ordinates but it wouldn't be BLY as those units.   By cosmological co-ordinates you're probably talking about co-moving co-ordinates and those have distances in fairly arbitrary units.
I didn't say comoving distance or comoving coordinates, I said proper distance. No light that we see today has ever been a greater proper distance from here than 6BLY. That means you're measuring the distance with a bunch of stationary (zero peculiar velocity) meter sticks end to end. So there very much are units to proper distance, and that was what I meant by that statement

Quote
1 unit of spatial separation in the co-moving frame is a time-dependant, ever increasing amount of actual physical distance.
Agree, I just wasn't using such units.

Quote
(Minor note,  I'm finding more references claiming the radius of the observable universe is approx. 46.5 BLY  and not 48 BLY - but it's not worth quibbling over).
I've always imagined that it was something very dependent on the current agreed upon values for the ratio of mass to dark energy, so I'm fine with the latest accepted figure.

My only comment to Aeris that that matter and antimatter are not inherently more attracted to each other. A hydrogen atom and an antihydrogen atom might have immeasurably small gravitational attraction between them, but that's it, same as two regular hydrogen atoms.
Title: Re: Even More Questions that Antimatter to Me
Post by: Aeris on 30/10/2021 13:25:38
Eternal Student

"Well, it does seem to be.    However, you shouldn't just believe everything I've said.  The earlier post pulls a collection of different articles and ideas together.   It's an attempt to answer a direct question   "Why does matter annihilate with anti-matter?"   using what information is available.  You should check the sources of information yourself, make your own decisions and carefully note where any caveats or direct menton of a lack of evidence has been mentioned.  For example, I mentioned that although there should be an anti-particle for every fundamental particle in the standard model of physics, we haven't observed all of these and the idea does remain theoretical."
   
"As Alancalverd indicated, many people don't ask why and indeed there aren't any articles I can find that attempt to explain why.  So what you've got is a collection of things we do know about related situations and these have been stitched together to try and answer your question in what is hopefully a logical and consistent way.   It's the best I can offer:  There just aren't any definitive articles or texts written to explain WHY it happens (well, not that I could find) although there are many theories in which it CAN happen.   Presumably you haven't found it yourself with Google and/or you found quite a lot of conflicting information."

Trust me, I have spent hours researching this stuff online and I would not be here asking this question if I didn't already find a satisfying answer anywhere else. My question's unbelievably specific as it is, so the fact that you were even able to give me a solid-enough answer is really quite impressive. No, I shouldn't just believe every word you say, but I honestly have very little reason to NOT believe a lot of what you so considering just how much more reliable you guys have been than other science forums I've visited in the past.

Halc

"But you still got it wrong, and I edited your post, putting my name on the words that were mine.
A good deal of the problem is that Aeris doesn't seem to be using the quote feature and is just putting other people's works in quotation marks which doesn't identify whose words they are."

Yeah, sorry about that. I'm only responding to answers the way I am because A) I'm responding to multiple people at once and this is quicker than creating 2-4 separate posts, and B) I admittedly don't know how to use the Quote option that well.

"My only comment to Aeris that that matter and antimatter are not inherently more attracted to each other. A hydrogen atom and an antihydrogen atom might have immeasurably small gravitational attraction between them, but that's it, same as two regular hydrogen atoms."

Interesting. Thanks for the info.

 
Title: Re: Even More Questions that Antimatter to Me
Post by: Origin on 30/10/2021 13:48:33
I admittedly don't know how to use the Quote option that well
All you have to do is highlight the part of the post you want to quote and then click on the blue "ACTIONS" box at the top right of the post and then click "Quote (selected)".   
Title: Re: Even More Questions that Antimatter to Me
Post by: Eternal Student on 30/10/2021 16:20:40
Hi again.

An electron and positron frequently form "positronium" for a short period before they annihilate.
- So annihilation is much more likely if the electron and positron are in the "same" location and velocity for some time before they annihilate.
   Thanks for the reply and the extra information.  I knew that positronium could sometimes be formed but not that it was frequently formed.
    It's still talking about annihilation as being "likely", i.e. some random process that takes a certain amount of time before it actually happens.

It takes many elastic collisions with electrons and atomic nuclei (emitting electromagnetic radiation) before the positron slows down enough to form positronium, after which it will annihilate with confidence.
    Taking the phrase "with confidence" to mean ... well not with any certainty at all..... just that it becomes more likely with increasing elapsed time (if I've understood the situation correctly).   Positronium has an average life-time that can be determined by experiment but there is variation between individual positronium atoms, it's a bit like other radioactive decay, a seemingly random process as far as we know.

But you still got it wrong, and I edited your post, putting my name on the words that were mine.
   Sorry and thanks.

I didn't say comoving distance or comoving coordinates, I said proper distance. No light that we see today has ever been a greater proper distance from here than 6BLY.
   OK, I think I see what you've done but I'm still not sure how you got the figure to be as low as 6 BLY.   It's certainly under 13.8 BLY.  Anyway, this is what I get:

Take co-moving co-ordinates where t = 0 is the singularity where the scale factor a(0) = 0.  Assume a photon was emitted by a distant star at some co-moving time tE  with  tE > 0.   We don't really care how far away the star is now or ever was.  Our focus is on the distance between us and the photon.  We will assume the photon has reached us now, at co-moving time to   with  to > tE.

 Star                    Photon                 Us (Earth)   
   *                        ~~>                        @   
                              <----------------------->    d(t) = Distance from photon to Earth at co-moving time t

So we want of find d(t) = the proper distance between us and the photon at a co-moving time, t      with    tE ≤ t ≤ to.

   The co-moving distance χ(t)   =   b1bd35f9f7be1d10c8978449d1af9792.gif   =  an evolving (decreasing) separation between the photon and us as time progresses, measured in the co-moving frame.  Note that the lower integral limit is t and not tE.  We are not interested in the distance covered from the time of emission at the star, only in the co-moving distance from wherever the photon is at our arbitary time t and us here on earth (the location of where it will be observed by us at time to).

   So that the proper distance between us and the photon at co-moving time t  is 
d(t) =  a(t) .  χ(t)   =     a(t) . b1bd35f9f7be1d10c8978449d1af9792.gif      (I think that formula actually requires k = curvature of the FRW universe to be zero but that is our current belief anyway).

Fix  t   with    tE ≤ t ≤ to      So that the lower integral limit and the quantity a(t) become constants.

   Then  a(t) . b1bd35f9f7be1d10c8978449d1af9792.gif   =   1aadd6bbf8137d1fd7c96f07de425a47.gif   
[Equation 1]
   

Without assuming an explicit form for a(t) in [equation 1] we can only make limited progress.
Throughout the integral   a(t) ≤ a(t')    since t' ≥ lower integral limt and we can reasonably assume the scale factor has been a monotonically increasing function since the big bang.

   Hence,  1aadd6bbf8137d1fd7c96f07de425a47.gif   ≤   d50cf645ea890f747c9c7f24dea5eb22.gif   =  c . [to - t]  ≤ c [to ]     since we have 0 ≤ t ≤ to

   The co-moving time, to, when the photon was observed by us cannot be greater than the age of the universe = approx. 13.8 BY.  So the proper distance between the photon and us is never greater than 13.8 BLY.

   We can un-fix t now  -  our result holds for arbitrary t with   tE ≤ t ≤ to.   So we have that the proper distance bewteen us (here on earth) and a photon traveling toward us was never greater than  13.8 BLY.
   I can't see how you get this down to 6 BLY unless you assume a(t) has a particular form and apply this in [equation 1].

Best Wishes.
Title: Re: Even More Questions that Antimatter to Me
Post by: Halc on 30/10/2021 21:12:13
I can't see how you get this down to 6 BLY unless you assume a(t) has a particular form and apply this in [equation 1].
I followed and agree with all you posted, but it seems a lot of work just to show that it can't be > 13.8.
I did it the easy way and just looked up the graphs drawn for the papers illustrating reasonably (15 years?) recent solutions to FLRW metric that best match observations.
(https://i.stack.imgur.com/6yzwk.jpg)
Same picture, the top being proper distance and the bottom being comoving distance.
The red curve is our past light cone, which gets at its widest about 5.8 Glr just before it crosses the Hubble sphere at t=~3.7 Gyr.
Light from the most distant star visible is the dotted line at comoving 31Glr which crosses our light cone at a proper distance of only about ~2.5 Gl, far closer than light emitted more recently.
Title: Re: Even More Questions that Antimatter to Me
Post by: Eternal Student on 31/10/2021 00:27:02
Hi again,

   Thanks for the reply @Halc    I hadn't thought to look at a diagram like that, it does save a bit of time.  It looks like their scale factor is almost a linear function of time    a(t) ≈  k.t   except at early times.   They probably found a better solution for a(t) by running the Friedmann equations with our best estimates of the current proportions of matter, radiation etc.
    Anyway, just taking a(t) ≈ k.t   in  what I previously called [equation 1]  gives a maximum proper distance of approx. 5.1 BLY  which occurrs at a time that is about 5.1 BY after the big bang.   That's close enough to your diagram.  Especially since when you said  "...it crosses the Hubble sphere at t=~7.5 Gyr..."  you meant  4 Gyr or something under 5.

   Sorry for the distraction, Aeris, none of this is too important for your original questions.

Best Wishes.
Title: Re: Even More Questions that Antimatter to Me
Post by: evan_au on 31/10/2021 01:38:58
Quote from: Aeris
matter and antimatter are not inherently more attracted to each other
It's true that a neutron and an anti-neutron are not inherently attracted to each other, because:
- Gravity is negligible for subatomic particles
- neutrons & anti-neutrons have no electric charge (I understand that there is a slight magnetic moment, but small)
- The strong nuclear force operates over a small distance (around the diameter of a Uranium nucleus)
- The weak nuclear force has a very small range

However, for charged particles/antiparticles, there is a very strong electrostatic attraction:
- electrons (-) strongly attract positrons (+), briefly forming positronium
- protons (+) strongly attract antiprotons (-), forming protonium, half life 1us
       - Apparently, the protonium "atom" spends much of its short lifetime smaller than the width of a Uranium nucleus, so the strong nuclear force is actually more significant than the electrostatic force.

I didn't know (but I suspected) that there was a protonium, and it is listed in Wikipedia...
https://en.wikipedia.org/wiki/Protonium
Title: Re: Even More Questions that Antimatter to Me
Post by: Halc on 31/10/2021 18:18:22
I hadn't thought to look at a diagram like that, it does save a bit of time.  It looks like their scale factor is almost a linear function of time    a(t) ≈  k.t   except at early times.
That it is. Wiki has a scalefactor chart shown below. The magenta line is the one accepted not long ago, giving the 48BLY radius and the 13.8 age. If new tunings get closer to more recent observations, updating the radius of OU, the current age is probably updated as well.
(https://upload.wikimedia.org/wikipedia/commons/thumb/6/60/Mplwp_universe_scale_evolution.svg/330px-Mplwp_universe_scale_evolution.svg.png)
The line is nearly linear up to the current time, except at early times. It matches the diagram I chose in the prior post.

Quote
Especially since when you said  "...it crosses the Hubble sphere at t=~7.5 Gyr..."  you meant  4 Gyr or something under 5.
Yea, I fixed that, even before you pointed out the error. Brain fart reading the numbers wrong.

Quote
Sorry for the distraction, Aeris, none of this is too important for your original questions.
Yea, we kind of got off track, but it came up discussing how far away antimatter would have to be to not notice it, and that would be the OU, not some closer boundary such as the event horizon.
I've posted elsewhere that I don't except the existence of state, unmeasured. If you accept that, it means there's no antimatter galaxies at all, but distant parts of the universe might be in superposition of having matter and having antimatter galaxies. If you don't accept that, then cause and effect can be separated by greater than light speed, and the the antimatter balance far further away than 100 BLY might have, in violation of the principle of locality, caused our part of the universe to be everywhere matter.

This is why I think we're not entirely off topic with the discussion.
Title: Re: Even More Questions that Antimatter to Me
Post by: evan_au on 31/10/2021 20:05:19
Quote from: Halc
distant parts of the universe might be in superposition of having matter and having antimatter galaxies
Please clarify this. I don't understand:
- A sustained superposition of matter and anti-matter. Normally they annihilate
- How conservation laws exist in such a superposition
- If it occurs far away, why wouldn't it occur here (and be seen in the LHC, for example?)



Title: Re: Even More Questions that Antimatter to Me
Post by: Halc on 01/11/2021 01:19:26
Quote from: Halc
distant parts of the universe might be in superposition of having matter and having antimatter galaxies
Please clarify this. I don't understand:
- A sustained superposition of matter and anti-matter. Normally they annihilate
No, they do that if they're both matter and antimatter. It isn't in that state any more than there is both a live and dead cat in the box.

Quote
How conservation laws exist in such a superposition
I don't think there's a conservation law with matter/antimatter since we're quite able to annihilate matter without first finding antimatter to die with it.
Given a closed system, I don't think it can be in superposition of say different total momentum, so I don't see how the superposition challenges any particular law.

Quote
If it occurs far away, why wouldn't it occur here (and be seen in the LHC, for example?)
No interpretation allows superposition to be self-detectable. Rovelli especially gets into that.
The answer to this is quite interpretation dependent, so maybe picking one is a good place to start.
Title: Re: Even More Questions that Antimatter to Me
Post by: Eternal Student on 01/11/2021 14:53:49
Hi again.

Regarding this part of the OP:
2. A lot of people say that there is a lot more matter in our universe than antimatter, but how certain can we really be about this? Like, are we absolutely certain that there is way more regular matter in our universe than antimatter? What’s wrong with the idea of the universe having equal amounts of both, albeit in places far apart from each other?
    I think people have already covered the basic ideas.   Although matter and anti-matter annihilations do seem to be a bit random, they do happen often and will be generally favoured or statistically likely.   As such, we would expect to see annihilation reactions going on where any pocket of anti-matter formed a boundary or border with a pocket of ordinary matter.  These annihilations would tend to produce gamma rays, which we just aren't observing.
    However, space is big and maybe we just haven't been looking in the right place.   One thing is that there just doesn't seem to be any reason why anti-matter would behave differently to ordinary matter, so it should be fairly evenly mixed up and distributed with ordinary matter.  There shouldn't be much reason for anti-matter to have attracted other anti-matter and ordinary matter to attract ordinary matter so that it would have separated into pockets of ordinary matter and pockets of anti-matter.  It should all be just fairly evenly mixed up.  If such pockets of ordinary matter and pockets of anti-matter exist then it is presumably just because over time there have been annihilations and there was a surplus of anti-matter or ordinary matter in those pockets to start with.  If this is just some random thing then we should be finding pockets of anti-matter all over the place.  In the simplest situation we should be finding some anti-matter in the particles that hit earth from outer space - but we don't seem to.   
    There are numerous articles and YT videos that discuss this issue and it is one of the main puzzles or problems with the mainstream models of cosmology that we have - why does there seem to have been a slight surplus of ordinary matter compared to anti-matter (at least in our observable region of space)?

    It's possible to imagine extreme situations where pockets of anti-matter might be kept quite separate from ordinary matter.  We can store anti-protons in magnetic bottles and prevent it from coming into contact with ordinary matter.  So it's possible to imagine some naturally occurring astronomical structures with magnetic fields that might act as giant bottles for anti-matter in the universe.  However this is speculation, I'm not aware of any such structures.
   If you assume that it is simply that the anti-matter is very far away from here, then the main problem is explaining why this peculiar distribution occurred during the evolution of the observable universe.  It could be just random chance but this chance would be incredibly small.  Antimatter should have been treated fairly equally with ordinary matter and we would expect them both to have been fairly evenly distributed.   However, if the universe is infinite then perhaps we can go with this random event occurring and argue along the lines of something like the anthropic principle.

Best Wishes.
Title: Re: Even More Questions that Antimatter to Me
Post by: Eternal Student on 03/11/2021 13:20:41
Hi again.

   I should probably have a look at the last remaining question in the OP.

3. Let’s say, hypothetically speaking (key word, hypothetically, NOT theoretically), I created a wooden, regular matter chair from a huge amount of energy without creating any antimatter whatsoever. It really doesn’t matter (no pun intended) how exactly I did this. All that matters is that I created lots of matter without creating even a little bit of antimatter. How many physical laws would I break from doing this?
    Well there has been some discussion of E=mc2  with at least one post talking about nuclear reactions where there is an overall mass deficit resulting in a large release of energy.  It's reasonable to assume that we can convert mass to/from energy.   Mass is just a property of some matter and it's surprisingly easy to adjust the mass of some piece of matter, we can just get it moving at relativistic speeds for example.  Alternatively we can just get some piece of matter very hot to increase its mass.  There is a difference between creating additional mass (which is easy) and actually creating an additional fundamental particle (which is hard). 

     We can put some photons into a box, we can have mirrors on the walls of box to keep the photons bouncing around inside.  We can put quite a lot of photons into the box if we want.  None the less, when we open the box and have a look inside there is usually just some photons in the box and not some particles of matter that have formed.  Energy density at some place does not force the formation of matter particles and vice versa  (particles of matter are under no obligation to change into some other form of energy).   E=mc2 is just a relationship between two quantities (mass and energy) and not a prescription for actually creating new matter particles.

     There are very few ways of creating new matter particles that we know about.  We believe that there was some particle synthesis shortly after the big bang.   Specifically, that two energetic photons can combine under the right circumstances and create a fundamental particle of matter like a quark or a lepton.   Additionally we believe that the weak nuclear interaction is capable of changing one type of fundamental particle into another.   That's it, that's the only two ways I know of to bring about a new fundamental particle type in a place where there wasn't one before.
    (Well, I suppose there's a third option.... we can distort spacetime sufficiently and see how this might affect the permitted solutions of quantum field theory in the vicinity.  This is the kind of thing that happens with Hawking radiation, where particles do appear just outside the event horizon as far as an observer outside of the black hole would be concerned.  I'm just going to ignore this and I'm hardly competent to talk about it anyway).

    Now, let's have a careful look at your (Aeris) original question.    "(...suppose...) I created a wooden, regular matter chair from a huge amount of energy without creating any antimatter whatsoever".    The good news is that this is quite conceivable and doesn't violate any laws of science at all.  Just remember that "energy" isn't any kind of substance anyway.   So your supply of energy could have been some matter that you found lying about the place (matter has energy or some people might say matter is a form of energy).   Given some hand tools, it's possible to create a wooden chair from some wood and you don't usually create any anti-matter while you're doing this.   Even if you didn't have any actual wood but just some other matter available, we can imagine being able to convert the matter you have into some carbon-based wood matter using the weak interaction.
    This is probably not what you (Aeris) meant at all.  When you stated that you were creating a wooden chair from energy you probably had in mind some supply of photons that you were going to use.  Well, that's where it gets a bit tricky.  The synthesis of quarks and leptons (fundamental particles of matter) from photons does seem to involve the creation of  particle and anti-particle pairs.   I think Halc already mentioned that you could just throw all the anti-matter away if you wanted (for example fire it into a black hole).  It's technically still there and would persist on the event horizon for eternity as far as an observer outside the black hole is concerned - but it's not going to get much opportunity to interact with anything.

Best Wishes.
     
Title: Re: Even More Questions that Antimatter to Me
Post by: Aeris on 03/11/2021 16:07:51
Eternal Student

Hi again. I'm so sorry I haven't been as active as I was last week. I've just returned to college and I've already been showered in homework. I don't have enough time to go through every else everyone has said since my last comment on this topic, so I'm just gonna respond to this one in particular since it's the one that interests me the most.

"We can put some photons into a box, we can have mirrors on the walls of box to keep the photons bouncing around inside.  We can put quite a lot of photons into the box if we want.  None the less, when we open the box and have a look inside there is usually just some photons in the box and not some particles of matter that have formed.  Energy density at some place does not force the formation of matter particles and vice versa  (particles of matter are under no obligation to change into some other form of energy).   E=mc2 is just a relationship between two quantities (mass and energy) and not a prescription for actually creating new matter particles."

"There are very few ways of creating new matter particles that we know about.  We believe that there was some particle synthesis shortly after the big bang.   Specifically, that two energetic photons can combine under the right circumstances and create a fundamental particle of matter like a quark or a lepton.   Additionally we believe that the weak nuclear interaction is capable of changing one type of fundamental particle into another.   That's it, that's the only two ways I know of to bring about a new fundamental particle type in a place where there wasn't one before.
    (Well, I suppose there's a third option.... we can distort spacetime sufficiently and see how this might affect the permitted solutions of quantum field theory in the vicinity.  This is the kind of thing that happens with Hawking radiation, where particles do appear just outside the event horizon as far as an observer outside of the black hole would be concerned.  I'm just going to ignore this and I'm hardly competent to talk about it anyway)."

I actually didn't know that's what E = MC² actually meant. I always thought it quite literally meant that energy could be turned into matter (AKA electrons, protons and neutrons). It does raise some interesting questions though. Namely A) When a nuclear bomb or the sun converts matter to energy, is it actually just converting the mass of its matter into energy or are particles like electrons and protons decaying? B) If it turns out that particles like electrons and protons DON'T decay and last forever, does that mean that matter itself is eternal and has always existed? and C) Is there any evidence for particle synthesis and is it a process that we could theoretically recreate on Earth (be it now or in the distant future)?

"Now, let's have a careful look at your (Aeris) original question.    "(...suppose...) I created a wooden, regular matter chair from a huge amount of energy without creating any antimatter whatsoever".    The good news is that this is quite conceivable and doesn't violate any laws of science at all.  Just remember that "energy" isn't any kind of substance anyway.   So your supply of energy could have been some matter that you found lying about the place (matter has energy or some people might say matter is a form of energy).   Given some hand tools, it's possible to create a wooden chair from some wood and you don't usually create any anti-matter while you're doing this.   Even if you didn't have any actual wood but just some other matter available, we can imagine being able to convert the matter you have into some carbon-based wood matter using the weak interaction.
    This is probably not what you (Aeris) meant at all.  When you stated that you were creating a wooden chair from energy you probably had in mind some supply of photons that you were going to use.  Well, that's where it gets a bit tricky.  The synthesis of quarks and leptons (fundamental particles of matter) from photons does seem to involve the creation of  particle and anti-particle pairs.   I think Halc already mentioned that you could just throw all the anti-matter away if you wanted (for example fire it into a black hole).  It's technically still there and would persist on the event horizon for eternity as far as an observer outside the black hole is concerned - but it's not going to get much opportunity to interact with anything."

Since particle synthesis is the closest of those three methods you said to what you assumed I was describing in my question (which you were absolutely right about btw well done), that's the method I will choose to explain this question more thoroughly. Obviously a machine with the ability to do what the Big Bang did, even on a much smaller scale, would most likely be huge in size and require a level of energy akin to... well I actually don't know but I have no doubt in my mind it's probably an insane amount, so to make this slightly easier to answer, let's just pretend for a moment we're dealing with technology belonging to a type 2 or 3 civilization. I was mainly interested interested in knowing about what physical, universal laws would prevent a theoretical process like this from working. A perpetual motion machine of the first kind wouldn't work due to the first law of thermodynamics, and a perpetual motion machine of the second kind wouldn't work due to the second and third law of thermodynamics. Something like generating a wormhole however, is theoretically possible on paper (not everyone agrees with this statement, but many of them do), even though we currently don't know how to do it, so would a process involving the creation of matter from energetic particles like photons without the simultaneous creation of antimatter be impossible, or possible in theory? At least two people have brought up the fact that conversation of charge is safe due to the creation of electrically-neutral molecules, but evan_au brought up the fact that I violated conversation of lepton and baryon number, which left me confused since, like, aren't those number already uneven due to the universe having more matter than antimatter in it? If this really is impossible, how about a way to turn antimatter into regular matter? Would that also be impossible?
Title: Re: Even More Questions that Antimatter to Me
Post by: Origin on 03/11/2021 17:56:58
I actually didn't know that's what E = MC² actually meant.
The equation 281a70c20b16a38d7781189936e1ac9f.gif shows the relationship between energy and matter.
Quote
I always thought it quite literally meant that energy could be turned into matter (AKA electrons, protons and neutrons).
Energy can literally be turned into matter.  A photon with > 1.02 MeV can produce an electron - positron pair.
Title: Re: Even More Questions that Antimatter to Me
Post by: Eternal Student on 04/11/2021 14:43:20
Hi.

The equation 281a70c20b16a38d7781189936e1ac9f.gif shows the relationship between energy and matter.
   This is fine as a turn of phrase and I'm sure that you know what it means, Origin but in this context where we're talking about creating new particles and explaining it to Aeris we have to take some care.   Matter has some mass but mass isn't a direct measure of how much matter is present.  (Inertial) Mass is the resistance to an applied force that would cause a change in momentum for an object.
    The  symbol  m  in  E=mc2  stands for mass and not  matter.   An amount of matter is measured in moles  (or mols) and this equation tells us nothing about how much energy is equivalent to one mole of substance.     Meanwhile mass is measured in Kilograms and is the only thing we can put into this equation.
    For example 1 mol of Hydrogen gas (H2) has a mass of 2 grams at typical temperatures.   However, if we get it hot then we add internal energy and the mass of the Hydrogen will increase, even though there are still Avogadros number of molecules present.  The amount of matter hasn't changed but the mass of it has.  This sometimes takes a moment to think about.  If you put some hydrogen into a container then it requires a certain amount of force to make it accelerate at 1 m/s/s .   If you heat up that Hydrogen then it really does require more force to achieve that same acceleration even though there is exactly the same amount of Hydrogen there.
   We can can add energy to increase the mass of something easily but creating additional particles is more complicated.

Energy can literally be turned into matter.  A photon with > 1.02 MeV can produce an electron - positron pair.
   Yes but not on it's own.   We only observe this pair production when the photon is in the vicinity of a dense nucleus.   This process is also quite random, we sometimes observe a positron and an electron being created but sometimes we don't.    (https://en.wikipedia.org/wiki/Pair_production).
   If we don't follow a prescription or procedure (like firing the photon into a dense nucleus) then the high energy photon will just sail on through space and remain as a photon.  It doesn't spontaneously change into a positron and an electron all on its own  Indeed, it cannot do this because there would be no way to conserve momentum in all frames of reference without a nucleus that can be given recoil momentum.  As such it becomes a bit arbitrary to say that the photon just spontaneously changed into a pair of particles of matter.  There was some interaction that required a nucleus to be there and the nucleus is given recoil momentum at the end.
    Finally note that a photon is not "energy", it is something that has energy.  Energy does not have to be any kind of substance (it's just a conserved quantity).  Changing something with energy into some particles of matter is one thing, there are some procedures for doing this.  However, getting some energy density in one place does not automatically create a fundamental particle of matter like a lepton or a quark at that place.  You usually just have some energy density at that place.  There is no rest mass (or inertial mass) you could measure there and no fundamental particle of matter you can find there.   This was the main point being described earlier.  E = mc2 does not imply that just getting some energy density in one place will automatically create a particle of matter, there's no obligation for one form of energy to spontaneously change into 'rest mass'. 

(already too long... I'm stopping).

Best Wishes.
Title: Re: Even More Questions that Antimatter to Me
Post by: Origin on 04/11/2021 15:13:16
This is fine as a turn of phrase and I'm sure that you know what it means, Origin but in this context where we're talking about creating new particles and explaining it to Aeris we have to take some care.
Yes, I meant to say mass, since that is what the m stands for.  It is an important distinction.
Yes but not on it's own.   We only observe this pair production when the photon is in the vicinity of a dense nucleus.
I never said what else was involved and I don't think it matters, the point was that photons can be directly converted into a electron and a positron. 
Title: Re: Even More Questions that Antimatter to Me
Post by: Eternal Student on 04/11/2021 16:22:40
Hi again.

I never said what else was involved and I don't think it matters, the point was that photons can be directly converted into a electron and a positron. 
    Yes, exactly -  and this relates to one of Aeris' questions in the most recent post....

C) Is there any evidence for particle synthesis and is it a process that we could theoretically recreate on Earth (be it now or in the distant future)?

1.   We can make positrons and electrons from gamma rays.  Just fire them toward some dense nuclei (like a lump of gold).   It's random but you get a fair chance for particle and anti-particle pair production if the nuclei have a high atomic number (like Gold) and also the gamma rays are of a frequency corresponding to an energy significantly above 1.022 MeV.

2.   There are also some reports where particle and anti-particle pairs have been created just by the collision of two photons  (i.e. without the need for a dense nucleus in the vicinity).   (see https://en.wikipedia.org/wiki/Matter_creation  but note that there are few credible sources and references on that article and I haven't read any of those myself).  It seems that this should happen and it is what we think must have happened in the early universe (since there wouldn't have been any dense nuclei around to get involved with matter creation until after there were already some nuclei).

3.   It is possible to create all fundamental particles in the standard model, including quarks, leptons and bosons using photons of varying energies above some minimum threshold, whether directly (by pair production), or by decay of the intermediate particle (such as a W− boson decaying to form an electron and an electron-antineutrino).  - quote from Wiki.   This is mainly a statement of theoretical possibility and I'm quite certain that we have NOT actually seen a great many heavy particles being created from photons in any experiments:   In no small part this is because the energies of the photons required are huge.  To create baryons like a proton and an anti-proton we should require photons in the hard gamma ray range,  > 1.88 GeV,  or  3 orders of magnitude higher than the experiments we have done to create electrons and positirons. 
    - - - - - -

It should be noted that in all of these particle synthesis reactions, we do seem to create everything in matter and anti-matter pairs.  This, along with the general theory suggests that quantities like "lepton number" and "baryon number" must be conserved.

Best Wishes.
Title: Re: Even More Questions that Antimatter to Me
Post by: Aeris on 04/11/2021 20:14:55
Eternal Student

"1.   We can make positrons and electrons from gamma rays.  Just fire them toward some dense nuclei (like a lump of gold).   It's random but you get a fair chance for particle and anti-particle pair production if the nuclei have a high atomic number (like Gold) and also the gamma rays are of a frequency corresponding to an energy significantly above 1.022 MeV."

"2.   There are also some reports where particle and anti-particle pairs have been created just by the collision of two photons  (i.e. without the need for a dense nucleus in the vicinity).   (see https://en.wikipedia.org/wiki/Matter_creation  but note that there are few credible sources and references on that article and I haven't read any of those myself).  It seems that this should happen and it is what we think must have happened in the early universe (since there wouldn't have been any dense nuclei around to get involved with matter creation until after there were already some nuclei)."

"3.   It is possible to create all fundamental particles in the standard model, including quarks, leptons and bosons using photons of varying energies above some minimum threshold, whether directly (by pair production), or by decay of the intermediate particle (such as a W− boson decaying to form an electron and an electron-antineutrino).  - quote from Wiki.   This is mainly a statement of theoretical possibility and I'm quite certain that we have NOT actually seen a great many heavy particles being created from photons in any experiments:   In no small part this is because the energies of the photons required are huge.  To create baryons like a proton and an anti-proton we should require photons in the hard gamma ray range,  > 1.88 GeV,  or  3 orders of magnitude higher than the experiments we have done to create electrons and positirons." 
    - - - - - -

"It should be noted that in all of these particle synthesis reactions, we do seem to create everything in matter and anti-matter pairs.  This, along with the general theory suggests that quantities like "lepton number" and "baryon number" must be conserved."

Ok neat, so this process is possible, but now I have 2 more questions I need you to take me through.

1. We can already create pairs of particles using this process, but can we potentially go a little further and create full-on atoms and molecules? Maybe some hydrogen gas or some water or something?
2. What are you referring to when you say when you say general theory?
3. When you say lepton number and baryon number must be conserved, what exactly do you mean by that. Are our current models of the universe dependent on those qualities being conserved, or will something terrible happen to the universe if they aren't conserved? Also, how are these numbers conserved? Is it like conservation of charge where the net amount of leptons and baryons in the entire universe is zero? Considering there's practically no antimatter in the entire universe, that seems quite unlikely to be the case, so what exactly is wrong with the idea of a process that results in the formation of only regular matter and no antimatter?   
Title: Re: Even More Questions that Antimatter to Me
Post by: Halc on 04/11/2021 22:09:57
1. We can already create pairs of particles using this process, but can we potentially go a little further and create full-on atoms and molecules? Maybe some hydrogen gas or some water or something?
They've already made anti-hydrogen, and have it stored in a fancy bottle.  Anything bigger requires fusion, and we already have such a hard time doing that with ordinary matter, it doesn't seem likely that they're going to make an anit-oxygen nucleus anytime soon.

Quote
When you say lepton number and baryon number must be conserved, what exactly do you mean by that. Are our current models of the universe dependent on those qualities being conserved, or will something terrible happen to the universe if they aren't conserved? Also, how are these numbers conserved? Is it like conservation of charge where the net amount of leptons and baryons in the entire universe is zero? Considering there's practically no antimatter in the entire universe, that seems quite unlikely to be the case, so what exactly is wrong with the idea of a process that results in the formation of only regular matter and no antimatter?   
Well obviously they're not conserved (not locally at least). This is a problem yet to be solved. So your assessment above is right. Maybe it has to do with some kind of imbalance during the inflation phase, where a random chance creation of matter or antimatter is multiplied by processes that don't obey conservation laws that have not yet been set up. This is a wild guess, undoubtedly wrong, but a solution to the problem will perhaps require thinking along such lines.
Maybe dark matter at sufficient densities has properties where matter or antimatter condense out, but not both.
Title: Re: Even More Questions that Antimatter to Me
Post by: alancalverd on 04/11/2021 22:28:45
It's important to remember that the "laws" of physics are descriptive, not prescriptive. They are how we observe the universe to work, not how some authority has ordained that it must work. So it's entirely possible that the creation of the observable universe did not follow some or indeed any of the laws we see in action every day, and even possible that we may occasionally observe some residue of the prior universe that seems to breach those laws.
Title: Re: Even More Questions that Antimatter to Me
Post by: Eternal Student on 05/11/2021 02:39:01
Hi again.
 
1. We can already create pairs of particles using this process, but can we potentially go a little further and create full-on atoms and molecules? Maybe some hydrogen gas or some water or something?
   You've had a good answer from Halc concerning creation of anti-matter atoms.
I'm suspicious you wanted an answer about creating ordinary matter atoms and/or entire molecules.  I'm also suspicious you have an eye on the science ficition ideas of creating matter in something like a start trek transporter or whatever the thing is they make stuff in...  a replicator I think - where Captain Picard gets his cup of Earl Grey.
    Well,  making an entire atom all in one go,  or straight from a photon is asking a lot but I suppose it's not impossible.   The energy of the photon(s) has to match or exceed the total rest mass of the particles you are hoping to create.  So for an entire  atom of hydrogen  plus  its partner  anti-hydrogen  that  works out at a photon with an energy of  1.88 GeV or more.   I haven't done that calculation... just found it on a web-page but it looks about right.   The protons and anti-protons are much more massive than an electron and positron which you could create with just 1.022 MeV.     Or to say it another way, the protons need  about 1800 times more energy in the gamma ray because they are about 1800 times more massive than an electron.    I really don't know if we have equipment to create gamma rays with that frequency, its pretty serious cosmic ray stuff.
     Anyway supposing you can get the photons (you know.... on the Google shopping channel it's amazing what you can get these days), you could try and fire those at some dense nuceli and hope for some of them to change into hydrogen plus anti-hydrogen.   Now these photons are so energetic that their penetration and absorption characteristics are quite different to the soft gamma rays that produced position-electron pairs.  These hard gamma rays will frequently just go through a gold target as if it wasn't there.   You'd hope to fire millions of these gamma rays at a gold target and only get a few to interact.   There's also very little you can do to stop other particles being created.  It's not certain that a proton and an electron would appear, you might just get a proton (and it's anti-proton) and the remaining energy of the photon might just give those protons more kinetic energy than usual.   To say that another way, it's not certain that all the energy of the incident photon would be changed into rest mass, some of it might be converted into kinetic energy for a smaller particle.   Anyway... the whole process has become extremely random.   If you get anything at all it'll be a shower of assorted stuff, some of which would have very high velocities and you'd probably never be able to capture it - but maybe a small portion of that stuff would actually be an entire atom of hydrogen (and anti-hydrogen or bits of anti-hydrogen that have fallen apart).
    CERN do not make their anti-hydrogen this way.  It's far more efficient to make the smaller components like positrons and anti-protons and then they almost self-assemble themselves into anti-hydrogen.  They don't even try to make the anti-protons straight from photons but instead they bombard a metal target with ordinary protons and collect some anti-protons that are produced from that.  Even the positrons aren't synthesised from photons, instead these apparently come from a radioactive sodium source that just emits positrons as part of it natural decay process.   All they do at CERN is collect these components, slow them down and allow them to come into contact inside a trap and then just wait for self-assembly into anti-atoms.    (Information source:  https://newscenter.lbl.gov/2010/11/17/antimatter-atoms/).
   In principle if you have a pair of anti-hydrogen atoms close together (and cold, i.e. at low velocities) chemistry should take over and they should self-assemble into a complete molecule of  anti-Hydrogen gas  ( I think the molecular formula has a bar over the H    like   this   e4b95ac101fee6faf1154db2ab85202d.gif   ).
    Anyway, just focusing on the oridnary matter and not the anti-matter.  It would be more efficient to just try and sythesise the protons and electrons (probably in seperate places) and then push those protons through a cloud of electrons.  Since they have opposite charges they will attract and self-assemble into atoms of hydrogen at-least a fair portion of the time.  We already know that chemisry can take over form here and two atoms of Hydogen in close proximity will self-assemble into a molecule of Hydrogen gas (provided they are cold, i.e. have fairly low velocities).
    It's a bit messy and extremely random with high energy gamma rays and high velocity particle fragments flying off all over the place, plus it's a bit of hybrid between sythesising some elementary particles and then just sitting back and waiting for ordinary physics and chemistry to build a molecule for you.   However, with a few years of refinement it might look a little bit more like the replicator where Captain Picard gets his cup of Earl Grey tea.

Best Wishes.
Title: Re: Even More Questions that Antimatter to Me
Post by: Eternal Student on 05/11/2021 13:31:20
Hi again.
2. What are you referring to when you say when you say general theory?
   I was being quite vague.   Most theories concerning the standard model of particle of physics (so that'll be Quantum Field theory mainly) state that the baryon number and lepton number is always conserved in any interaction.     (See, https://en.wikipedia.org/wiki/Baryon_number,  for example).
     The synthesis of matter from photons seems to do this by always creating matter and anti-matter pairs, whenever the process has been observed in experiments.
   
3. When you say lepton number and baryon number must be conserved, what exactly do you mean by that.
   For a system the Baryon number, B, is a quantum number defined by
B = 53ea53f956166447a8835c9e134e6160.gif
    Where n1 = the number of quarks in the system    and  n2 = the number of anti-quarks in the system.

Similarly Lepton number is   L = N1  -  N2   where  N1   and  N2 are the numbers of Leptons and anti-Leptons  respectively.

    Along with  statements like this:
In particle physics, lepton number (historically also called lepton charge)[1] is a conserved quantum number representing the difference between the number of leptons and the number of antileptons in an elementary particle reaction.[2] Lepton number is an additive quantum number, so its sum is preserved in interactions     - Taken from https://en.wikipedia.org/wiki/Lepton_number.

Examples include radioactive decay by  Beta-  emission.
   n    →     p+  +  e-   +  1f6de224176a0519faaafe78915746bb.gif

Initially (on the left hand side) we had a neutron,      B = 1  and no leptons   L =0.
After the interaction we have a proton   B =1   and an electron (L=+1) but also an  electron anti-neutrino  (L = -1)  giving a total  L = 1 - 1 = 0  = exactly the same as we started with.
Is it like conservation of charge where the net amount of leptons and baryons in the entire universe is zero?
   Yes.  See above.   Technically, the conservation of lepton and baryon number only implies that whatever these numbers were to start with, they never change after any interaction.   So  they would be exactly 0 all the time    if and only if   they were 0 initially.   However if there were some leptons and baryons around just after the big bang then that is the total number that will be conserved from then on.    (We think that there was only radiation around just after the big bang and leptogensis and baryogensis happened shortly afterwards).   
Are our current models of the universe dependent on those qualities being conserved, or will something terrible happen to the universe if they aren't conserved?
   Conservation of these quantum numbers helps to explain why we don't observe some interactions. 

Lepton number was introduced in 1953 to explain the absence of reactions such as
    ν    + n   →    p  +    e−
   - Taken from Wiki.

   However, I'm not sure that it actually arises as a necessary condition from some piece of mathematics that describes the quantum mechanics.  It just seems to be an assumption in the standard model of physics and it is is empirically or experimentally supported (but I don't know for certain).
   Certainly there are some theories that suggest Lepton number and Baryon number do not need to be conserved.
   See:    "Violations of Lepton number conservation laws"   https://en.wikipedia.org/wiki/Lepton_number#Violations_of_the_lepton_number_conservation_laws   
  and also,    "Physics beyond the standard model"      https://en.wikipedia.org/wiki/Physics_beyond_the_Standard_Model

Will something terrible happen to our universe if they aren't conserved?  Well, the universe will presumably carry on doing what it's been doing regardless of whether we understand it or not.   We (human beings) will drop the assumption that Lepton number and Baryon number is necessarily conserved but I don't think it will shake the usefullness of the standard model of particle physics too much.  Under typical situations the standard model does seem to be a useful approximation or model even if there are some unusual situations (perhaps in the time close to the big bang) where it's not so good.
    As people have already mentioned, we do believe that there is more matter in our observable universe than anti-matter and so we are all expecting that there is at least some mechanism whereby lepton and baryon numbers are not conserved.
Considering there's practically no antimatter in the entire universe, that seems quite unlikely to be the case, so what exactly is wrong with the idea of a process that results in the formation of only regular matter and no antimatter?
   Many physicists do expect that there is (or was) such a process, or alternatively some process whereby anti-matter decays (presumably back into photons) faster or more preferentially than ordinary matter would undergo the same process.   It may have required conditions that were only around just after the big bang, so it may not be repeatable now.

Best Wishes.
Title: Re: Even More Questions that Antimatter to Me
Post by: Aeris on 05/11/2021 15:56:48
Eternal Student

"For a system the Baryon number, B, is a quantum number defined by
B = 53ea53f956166447a8835c9e134e6160.gif
    Where n1 = the number of quarks in the system    and  n2 = the number of anti-quarks in the system.

Similarly Lepton number is   L = N1  -  N2   where  N1   and  N2 are the numbers of Leptons and anti-Leptons  respectively.

    Along with  statements like this:
In particle physics, lepton number (historically also called lepton charge)[1] is a conserved quantum number representing the difference between the number of leptons and the number of antileptons in an elementary particle reaction.[2] Lepton number is an additive quantum number, so its sum is preserved in interactions     - Taken from https://en.wikipedia.org/wiki/Lepton_number.

Examples include radioactive decay by  Beta-  emission.
   n    →     p+  +  e-   +  1f6de224176a0519faaafe78915746bb.gif

Initially (on the left hand side) we had a neutron,      B = 1  and no leptons   L =0.
After the interaction we have a proton   B =1   and an electron (L=+1) but also an  electron anti-neutrino  (L = -1)  giving a total  L = 1 - 1 = 0  = exactly the same as we started with.
   Yes.  See above.   Technically, the conservation of lepton and baryon number only implies that whatever these numbers were to start with, they never change after any interaction.   So  they would be exactly 0 all the time    if and only if   they were 0 initially.   However if there were some leptons and baryons around just after the big bang then that is the total number that will be conserved from then on.    (We think that there was only radiation around just after the big bang and leptogensis and baryogensis happened shortly afterwards)."

Ah, I get it now. The numbers have been set in stone and they (to our knowledge at least) cannot change.   

"Conservation of these quantum numbers helps to explain why we don't observe some interactions. 

Lepton number was introduced in 1953 to explain the absence of reactions such as
    ν    + n   →    p  +    e−
   - Taken from Wiki.

   However, I'm not sure that it actually arises as a necessary condition from some piece of mathematics that describes the quantum mechanics.  It just seems to be an assumption in the standard model of physics and it is is empirically or experimentally supported (but I don't know for certain).
   Certainly there are some theories that suggest Lepton number and Baryon number do not need to be conserved.
   See:    "Violations of Lepton number conservation laws"   https://en.wikipedia.org/wiki/Lepton_number#Violations_of_the_lepton_number_conservation_laws   
  and also,    "Physics beyond the standard model"      https://en.wikipedia.org/wiki/Physics_beyond_the_Standard_Model

Will something terrible happen to our universe if they aren't conserved?  Well, the universe will presumably carry on doing what it's been doing regardless of whether we understand it or not.   We (human beings) will drop the assumption that Lepton number and Baryon number is necessarily conserved but I don't think it will shake the usefullness of the standard model of particle physics too much.  Under typical situations the standard model does seem to be a useful approximation or model even if there are some unusual situations (perhaps in the time close to the big bang) where it's not so good.
    As people have already mentioned, we do believe that there is more matter in our observable universe than anti-matter and so we are all expecting that there is at least some mechanism whereby lepton and baryon numbers are not conserved."

Yeah, I'll be honest, the way I worded that question made it sound like I was horrified at the prospect of the universe destroying itself through the violation of a fundamental law (or something pseudoscientific like that). I was primarily interested in knowing though since A) It's quite common for a single scientific discovery to overthrow and revamp entire models, and B) According to this https://www.quora.com/What-would-happen-if-the-law-of-conservation-of-charge-were-violated some conservation laws could cause VERY noticeable changes in our universe if they were ever violated and I was curious to know of the implications of a violation of baryon and lepton number. Actually, almost all of my questions stem from a place of curiosity, although I don't blame you for assuming I was interested in science-fiction (which admittedly I am, but that's not why I'm asking these questions).     

"Many physicists do expect that there is (or was) such a process, or alternatively some process whereby anti-matter decays (presumably back into photons) faster or more preferentially than ordinary matter would undergo the same process.   It may have required conditions that were only around just after the big bang, so it may not be repeatable now."

It's highly possible you'll respond to this with a "We don't know", but... do you think you could tell me what exactly those conditions were by any chance? Could you at least tell me what we think those conditions were?

Also this isn't relevant to your recent posts, but it is something that just crossed my mind earlier today that I didn't realize two days ago when you brought up particle synthesis. You said that "two energetic photons can combine under the right circumstances and create a fundamental particle of matter like a quark or a lepton.". You also said that such a process happened moments after the Big Bang. If matter didn't exist yet at this point in time, that means that electromagnetic fields didn't exist either. Photons are vibrations of electromagnetic fields, so how can photons exist moments after the big bang if electromagnetic fields did not?

Also not relevant to your recent posts, I'm having a fun time talking about this stuff with you guys :) you're really helping me out.   
Title: Re: Even More Questions that Antimatter to Me
Post by: Eternal Student on 05/11/2021 22:43:38
Hi again.

do you think you could tell me what exactly those conditions were by any chance? Could you at least tell me what we think those conditions were?
    Around the time of the big bang we expect the temperature, pressure and energy density to be extraordinarily high.  Physicist's often lump all of these descriptions and properties together and just say they are high energy conditions.  There may be other conditions like the inflaton field having only just switched off and/or the Higgs field having only just switched on (i.e. fields that we don't seem to be able to influence at the moment).
    There doesn't seem to be the technology available to recreate and also safely contain these extremes conditions and there are some theoretical limits anyway:  Since energy density and pressure are both sources of gravitation we could easily create a black hole by accident  (which would obviously tend to conceal what had actually happened during the experiment).   We are talking about establishing conditions that are within a hairs breadth of causing a singularity.   After all, baryogenesis and leptogenesis were thought to have occurred only a few moments after a singularity.
    At these sorts of energies it's thought that most of physics would be quite different.   Most of the theories we have are thought to be only low-energy approximations to the underlying principles.

If matter didn't exist yet at this point in time, that means that electromagnetic fields didn't exist either.
    That's not necessary.   The electromagnetic field exists and permeates all of space,  independently of whether there is matter there or not.   For example, an electric field exists even across a vaccum and any charged particle you had on the one side of that vaccum would still feel a force from a source on the other side of that vaccum.   EM radiation can certainly exist in and travel through a vaccum (infact it only has the speed c when it is doing this).
  (Technically the electromagnetic field is terminology associated with a classical field theory of electromagnetism and at these sorts of energies and relativistic conditions we should probably be using a quantised version but the principle would be the same.  Matter is not required).

Best Wishes.
I'm having a fun time talking about this stuff with you guys :) you're really helping me out.
   Great.  You're going to be helping me out in a few years and probably not just with some physics.

Best Wishes.
Title: Re: Even More Questions that Antimatter to Me
Post by: Aeris on 06/11/2021 20:21:53
Eternal Student

"Around the time of the big bang we expect the temperature, pressure and energy density to be extraordinarily high.  Physicist's often lump all of these descriptions and properties together and just say they are high energy conditions.  There may be other conditions like the inflaton field having only just switched off and/or the Higgs field having only just switched on (i.e. fields that we don't seem to be able to influence at the moment).
    There doesn't seem to be the technology available to recreate and also safely contain these extremes conditions and there are some theoretical limits anyway:  Since energy density and pressure are both sources of gravitation we could easily create a black hole by accident  (which would obviously tend to conceal what had actually happened during the experiment).   We are talking about establishing conditions that are within a hairs breadth of causing a singularity.   After all, baryogenesis and leptogenesis were thought to have occurred only a few moments after a singularity.
    At these sorts of energies it's thought that most of physics would be quite different.   Most of the theories we have are thought to be only low-energy approximations to the underlying principles."

Sooooo... the asymmetry that allowed regular matter to triumph over antimatter was brought about by the early universe being in a state of insanely high energy? It's that simple? Also theoretically speaking, if we COULD find a way to replicate this process, how much matter could we create before we unintentionally spawn a black hole? 

"That's not necessary.   The electromagnetic field exists and permeates all of space,  independently of whether there is matter there or not.   For example, an electric field exists even across a vaccum and any charged particle you had on the one side of that vaccum would still feel a force from a source on the other side of that vaccum.   EM radiation can certainly exist in and travel through a vaccum (infact it only has the speed c when it is doing this).
  (Technically the electromagnetic field is terminology associated with a classical field theory of electromagnetism and at these sorts of energies and relativistic conditions we should probably be using a quantised version but the principle would be the same.  Matter is not required)."

Ah, I see. So even if we found a way to magically remove the virtual particles popping in and out of existence all of the time, we would still have underlying, universal fields in space representing each of the four fundamental forces that hold all of existence together, thus preventing it from ever being truly empty. That also means that photons (and all forms of electromagnetic radiation) can exist independently of matter. Sweet!   

"Great.  You're going to be helping me out in a few years and probably not just with some physics."

Considering that you are the one researching, simplifying and explaining this stuff to me, it seems highly unlikely that, if I ever did take up a career in the world of science and physics, you'd be one of the people I'd teach. But still, thank you for the kind remark :)

Sorry for the late reply btw. Twas my Dad's birthday today.
Title: Re: Even More Questions that Antimatter to Me
Post by: Eternal Student on 06/11/2021 21:58:14
Hi.

Happy birthday to your dad.

Sooooo... the asymmetry that allowed regular matter to triumph over antimatter was brought about by the early universe being in a state of insanely high energy? It's that simple?
   I don't KNOW that for certain.  No one does at this time.   However, it seems likely.
You're partially ignoring the state of the Inflaton field and the Higgs field.   Specifically inflation (if it occured, which most physicists think it did) had only recently stopped in the early universe when matter synthesis was happening.   So the energy in the inflaton field had fallen but possibly not as low as it is now.  I'm not aware of any experiments that claim to influence the inflaton field.  Simply getting to high energies (like high velocity particles that might locally re-create conditions of high temperature and pressure) may not be enough to set the inflaton field back up to to the state it was in.

Also theoretically speaking, if we COULD find a way to replicate this process, how much matter could we create before we unintentionally spawn a black hole?
   I don't do these experiments, they are a bit beyond the tools I have in the kitchen.    There was some media hype a little while back about the possibility of accidentally creating a micro-black hole at CERN.   Of course the media didn't really say that it COULD create be a MICRO black hole   only that CERN could create a Black Hole.
    Here's some relevant web-pages:
https://scoolmedia.com/en/cern-creates-a-black-hole-that-they-can-not-control/
   Here's one quote I quite like......
...Scientists did succeed in making a black hole and
they could not control it. The microscopic black hole dropped right through the particle
accelerators containment fields, gravity pulled the hole towards the center of the Earth where
the hole now is growing bigger by absorbing the atoms surrounding it. It is growing slowly now
but with every minute passed it is getting larger and larger at an ever increasing pace. A few
days are left until the hole will grow big enough and two jets from two opposite sides of the
Earth will blow up and prove us that there is a giant black hole eating our world from the inside
out.


    This is the somewhat more credible version.   It's not that different, I suppose....
https://science.nasa.gov/science-news/science-at-nasa/2008/10oct_lhc
The world did not end. Switching on the world's largest and most powerful particle accelerator near Geneva, Switzerland, did not trigger the creation of a microscopic black hole. And that black hole did not start rapidly sucking in surrounding matter faster and faster until it devoured the Earth, as sensationalist news reports had suggested it might.

Anyway....  the important thing to note is that creating a microscopic black hole isn't as dramatic as creating a science fiction sized black hole.   Hawking radiation increases in rate as the size of black hole decreases,  so microscopic black holes should last only a fraction of second and they are only harmful in the sense of creating huge tidal forces in a region that is extremely close to them.    If a black hole formed that had the mass parameter of a particle,   then it creates no more gravitational pull on you (at a distance of say 5 metres) then the original particle had.   Let's phrase that another way,  we all experience some gravity on the surface of earth.   However we could replace all of that matter with a tiny black hole right where the centre of the earth was.  If that black hole has the same mass parameter (e.g. if it was made from the stuff the earth was made from but just compressed into a small size).   Then provided you find a way to stay a distance away from the black hole that is equal to the radius of the earth you wouldn't notice any difference in the force of gravity.
    Anyway,   CERN were hoping to observe interesting things that might even include a micro black hole   BUT  to the best of my knowledge, they haven't.

    So, I don't really know how to answer your question:  We could spawn lots of black holes but it doesn't necessarily need to be too much of problem.   We are getting close to the sorts of energy densities that should create a micro black hole.  The main problem is likely to be that the black holes would rip the particles created in the close region into .... only speculation knows.... and/or merge with them to make a larger hole for a brief moment.   So it could be more of a production problem rather than anything else:   Do you recall A level organic chemistry where the yield of the product you wanted to create is lower than you hoped for?  Usually because the conditions were wrong - like it was too hot and some other products were formed and/or some of the desired product was accidentally decomposed.   Anyway, a blackhole could be a bit like this - something ruining your yield of desired product.

Best Wishes.
Title: Re: Even More Questions that Antimatter to Me
Post by: Eternal Student on 06/11/2021 23:51:58
Hi again,

Ah, I see. So even if we found a way to magically remove the virtual particles popping in and out of existence all of the time, we would still have underlying, universal fields in space representing each of the four fundamental forces that hold all of existence together, thus preventing it from ever being truly empty. That also means that photons (and all forms of electromagnetic radiation) can exist independently of matter. Sweet!
    Yes, that's the right idea.
    Sadly, there's some junk or technical details that get in the way.  We couldn't remove all the virtual particles, they are a consequence of the field existing and quantum uncertainty.  So the only way to remove the virtual particles would be to remove the field.   Also, it's not certain that the four fundamental forces as we know them would exist in exactly the same way during the early universe.  The electromagnetic force seems to unify with the weak force to become the electro-weak force, for example.  So some of the fundamental forces are "missing" or blended into one.    However, you do seem to be expressing the general idea correctly.   Don't forget that matter particles like leptons and quarks should also be represented by their own fields (these particles are just excitations in their own underlying field, which just like other fields also exist throughout all of space).  So all we're really saying is that a photon field could have excitations in it (photons can exist) without any requirement for the matter fields to have some excitation (no matter particles have to exist).

it seems highly unlikely that, if I ever did take up a career in the world of science and physics, you'd be one of the people I'd teach.
    In a few years, if I can still work a computer, I will be needing help and some tolerance from people like yourself.

Best Wishes.